L'Hôpital's rule - How solve this limit question











up vote
0
down vote

favorite













How to solve this ?
$$lim_{xto 0} f(x);text{where};f(x)=frac{ arctan(2x)}{ln (x)}$$




The answer is $0$. My question is when we plug in $0$ in $f(x)$, we get the form $frac{0}{infty}$, which is not an indeterminate form, so we might just write $0$ as answer directly OR if we apply L'Hôpital's rule, we would still get an answer as $0$. Which method is correct?










share|cite|improve this question









New contributor




Amogh Joshi is a new contributor to this site. Take care in asking for clarification, commenting, and answering.
Check out our Code of Conduct.




















  • We can't use L'Hôpital's rule for the reasons you mentioned, so that approach isn't correct. You are essentially right with the first explanation. To see this more clearly think of $f(x) = g(x)h(x)$, where $g(x) = arctan(2x)$, and $h(x) = 1/ln(x)$. Do you know how to do these two limits separately?
    – Jabbath
    yesterday












  • First one because L-Hopital's rule is only applicable when you get an indeterminate form. You can get different answers if the limits are not indeterminate.
    – Sri Krishna Sahoo
    yesterday















up vote
0
down vote

favorite













How to solve this ?
$$lim_{xto 0} f(x);text{where};f(x)=frac{ arctan(2x)}{ln (x)}$$




The answer is $0$. My question is when we plug in $0$ in $f(x)$, we get the form $frac{0}{infty}$, which is not an indeterminate form, so we might just write $0$ as answer directly OR if we apply L'Hôpital's rule, we would still get an answer as $0$. Which method is correct?










share|cite|improve this question









New contributor




Amogh Joshi is a new contributor to this site. Take care in asking for clarification, commenting, and answering.
Check out our Code of Conduct.




















  • We can't use L'Hôpital's rule for the reasons you mentioned, so that approach isn't correct. You are essentially right with the first explanation. To see this more clearly think of $f(x) = g(x)h(x)$, where $g(x) = arctan(2x)$, and $h(x) = 1/ln(x)$. Do you know how to do these two limits separately?
    – Jabbath
    yesterday












  • First one because L-Hopital's rule is only applicable when you get an indeterminate form. You can get different answers if the limits are not indeterminate.
    – Sri Krishna Sahoo
    yesterday













up vote
0
down vote

favorite









up vote
0
down vote

favorite












How to solve this ?
$$lim_{xto 0} f(x);text{where};f(x)=frac{ arctan(2x)}{ln (x)}$$




The answer is $0$. My question is when we plug in $0$ in $f(x)$, we get the form $frac{0}{infty}$, which is not an indeterminate form, so we might just write $0$ as answer directly OR if we apply L'Hôpital's rule, we would still get an answer as $0$. Which method is correct?










share|cite|improve this question









New contributor




Amogh Joshi is a new contributor to this site. Take care in asking for clarification, commenting, and answering.
Check out our Code of Conduct.












How to solve this ?
$$lim_{xto 0} f(x);text{where};f(x)=frac{ arctan(2x)}{ln (x)}$$




The answer is $0$. My question is when we plug in $0$ in $f(x)$, we get the form $frac{0}{infty}$, which is not an indeterminate form, so we might just write $0$ as answer directly OR if we apply L'Hôpital's rule, we would still get an answer as $0$. Which method is correct?







limits






share|cite|improve this question









New contributor




Amogh Joshi is a new contributor to this site. Take care in asking for clarification, commenting, and answering.
Check out our Code of Conduct.











share|cite|improve this question









New contributor




Amogh Joshi is a new contributor to this site. Take care in asking for clarification, commenting, and answering.
Check out our Code of Conduct.









share|cite|improve this question




share|cite|improve this question








edited yesterday









Chinnapparaj R

4,4101725




4,4101725






New contributor




Amogh Joshi is a new contributor to this site. Take care in asking for clarification, commenting, and answering.
Check out our Code of Conduct.









asked yesterday









Amogh Joshi

183




183




New contributor




Amogh Joshi is a new contributor to this site. Take care in asking for clarification, commenting, and answering.
Check out our Code of Conduct.





New contributor





Amogh Joshi is a new contributor to this site. Take care in asking for clarification, commenting, and answering.
Check out our Code of Conduct.






Amogh Joshi is a new contributor to this site. Take care in asking for clarification, commenting, and answering.
Check out our Code of Conduct.












  • We can't use L'Hôpital's rule for the reasons you mentioned, so that approach isn't correct. You are essentially right with the first explanation. To see this more clearly think of $f(x) = g(x)h(x)$, where $g(x) = arctan(2x)$, and $h(x) = 1/ln(x)$. Do you know how to do these two limits separately?
    – Jabbath
    yesterday












  • First one because L-Hopital's rule is only applicable when you get an indeterminate form. You can get different answers if the limits are not indeterminate.
    – Sri Krishna Sahoo
    yesterday


















  • We can't use L'Hôpital's rule for the reasons you mentioned, so that approach isn't correct. You are essentially right with the first explanation. To see this more clearly think of $f(x) = g(x)h(x)$, where $g(x) = arctan(2x)$, and $h(x) = 1/ln(x)$. Do you know how to do these two limits separately?
    – Jabbath
    yesterday












  • First one because L-Hopital's rule is only applicable when you get an indeterminate form. You can get different answers if the limits are not indeterminate.
    – Sri Krishna Sahoo
    yesterday
















We can't use L'Hôpital's rule for the reasons you mentioned, so that approach isn't correct. You are essentially right with the first explanation. To see this more clearly think of $f(x) = g(x)h(x)$, where $g(x) = arctan(2x)$, and $h(x) = 1/ln(x)$. Do you know how to do these two limits separately?
– Jabbath
yesterday






We can't use L'Hôpital's rule for the reasons you mentioned, so that approach isn't correct. You are essentially right with the first explanation. To see this more clearly think of $f(x) = g(x)h(x)$, where $g(x) = arctan(2x)$, and $h(x) = 1/ln(x)$. Do you know how to do these two limits separately?
– Jabbath
yesterday














First one because L-Hopital's rule is only applicable when you get an indeterminate form. You can get different answers if the limits are not indeterminate.
– Sri Krishna Sahoo
yesterday




First one because L-Hopital's rule is only applicable when you get an indeterminate form. You can get different answers if the limits are not indeterminate.
– Sri Krishna Sahoo
yesterday










2 Answers
2






active

oldest

votes

















up vote
1
down vote













First of all, the limit cannot be $xto 0$, it must be $xto 0^+$, because of the domain of ln(x).



Second thing is that the L-Hopital rule is not applicable in this case, because it applies only for $frac{0}{0} or frac{infty}{infty}$ form, so the first method is correct.



Hope it is helpful.






share|cite|improve this answer




























    up vote
    0
    down vote













    Note that we can only consider $lim_{xto 0^+} f(x)$ and that we have a $frac 0{-infty}$ form which is not indeterminate.






    share|cite|improve this answer





















      Your Answer





      StackExchange.ifUsing("editor", function () {
      return StackExchange.using("mathjaxEditing", function () {
      StackExchange.MarkdownEditor.creationCallbacks.add(function (editor, postfix) {
      StackExchange.mathjaxEditing.prepareWmdForMathJax(editor, postfix, [["$", "$"], ["\\(","\\)"]]);
      });
      });
      }, "mathjax-editing");

      StackExchange.ready(function() {
      var channelOptions = {
      tags: "".split(" "),
      id: "69"
      };
      initTagRenderer("".split(" "), "".split(" "), channelOptions);

      StackExchange.using("externalEditor", function() {
      // Have to fire editor after snippets, if snippets enabled
      if (StackExchange.settings.snippets.snippetsEnabled) {
      StackExchange.using("snippets", function() {
      createEditor();
      });
      }
      else {
      createEditor();
      }
      });

      function createEditor() {
      StackExchange.prepareEditor({
      heartbeatType: 'answer',
      convertImagesToLinks: true,
      noModals: true,
      showLowRepImageUploadWarning: true,
      reputationToPostImages: 10,
      bindNavPrevention: true,
      postfix: "",
      imageUploader: {
      brandingHtml: "Powered by u003ca class="icon-imgur-white" href="https://imgur.com/"u003eu003c/au003e",
      contentPolicyHtml: "User contributions licensed under u003ca href="https://creativecommons.org/licenses/by-sa/3.0/"u003ecc by-sa 3.0 with attribution requiredu003c/au003e u003ca href="https://stackoverflow.com/legal/content-policy"u003e(content policy)u003c/au003e",
      allowUrls: true
      },
      noCode: true, onDemand: true,
      discardSelector: ".discard-answer"
      ,immediatelyShowMarkdownHelp:true
      });


      }
      });






      Amogh Joshi is a new contributor. Be nice, and check out our Code of Conduct.










       

      draft saved


      draft discarded


















      StackExchange.ready(
      function () {
      StackExchange.openid.initPostLogin('.new-post-login', 'https%3a%2f%2fmath.stackexchange.com%2fquestions%2f2997865%2flh%25c3%25b4pitals-rule-how-solve-this-limit-question%23new-answer', 'question_page');
      }
      );

      Post as a guest
































      2 Answers
      2






      active

      oldest

      votes








      2 Answers
      2






      active

      oldest

      votes









      active

      oldest

      votes






      active

      oldest

      votes








      up vote
      1
      down vote













      First of all, the limit cannot be $xto 0$, it must be $xto 0^+$, because of the domain of ln(x).



      Second thing is that the L-Hopital rule is not applicable in this case, because it applies only for $frac{0}{0} or frac{infty}{infty}$ form, so the first method is correct.



      Hope it is helpful.






      share|cite|improve this answer

























        up vote
        1
        down vote













        First of all, the limit cannot be $xto 0$, it must be $xto 0^+$, because of the domain of ln(x).



        Second thing is that the L-Hopital rule is not applicable in this case, because it applies only for $frac{0}{0} or frac{infty}{infty}$ form, so the first method is correct.



        Hope it is helpful.






        share|cite|improve this answer























          up vote
          1
          down vote










          up vote
          1
          down vote









          First of all, the limit cannot be $xto 0$, it must be $xto 0^+$, because of the domain of ln(x).



          Second thing is that the L-Hopital rule is not applicable in this case, because it applies only for $frac{0}{0} or frac{infty}{infty}$ form, so the first method is correct.



          Hope it is helpful.






          share|cite|improve this answer












          First of all, the limit cannot be $xto 0$, it must be $xto 0^+$, because of the domain of ln(x).



          Second thing is that the L-Hopital rule is not applicable in this case, because it applies only for $frac{0}{0} or frac{infty}{infty}$ form, so the first method is correct.



          Hope it is helpful.







          share|cite|improve this answer












          share|cite|improve this answer



          share|cite|improve this answer










          answered yesterday









          Crazy for maths

          4887




          4887






















              up vote
              0
              down vote













              Note that we can only consider $lim_{xto 0^+} f(x)$ and that we have a $frac 0{-infty}$ form which is not indeterminate.






              share|cite|improve this answer

























                up vote
                0
                down vote













                Note that we can only consider $lim_{xto 0^+} f(x)$ and that we have a $frac 0{-infty}$ form which is not indeterminate.






                share|cite|improve this answer























                  up vote
                  0
                  down vote










                  up vote
                  0
                  down vote









                  Note that we can only consider $lim_{xto 0^+} f(x)$ and that we have a $frac 0{-infty}$ form which is not indeterminate.






                  share|cite|improve this answer












                  Note that we can only consider $lim_{xto 0^+} f(x)$ and that we have a $frac 0{-infty}$ form which is not indeterminate.







                  share|cite|improve this answer












                  share|cite|improve this answer



                  share|cite|improve this answer










                  answered yesterday









                  gimusi

                  84.3k74292




                  84.3k74292






















                      Amogh Joshi is a new contributor. Be nice, and check out our Code of Conduct.










                       

                      draft saved


                      draft discarded


















                      Amogh Joshi is a new contributor. Be nice, and check out our Code of Conduct.













                      Amogh Joshi is a new contributor. Be nice, and check out our Code of Conduct.












                      Amogh Joshi is a new contributor. Be nice, and check out our Code of Conduct.















                       


                      draft saved


                      draft discarded














                      StackExchange.ready(
                      function () {
                      StackExchange.openid.initPostLogin('.new-post-login', 'https%3a%2f%2fmath.stackexchange.com%2fquestions%2f2997865%2flh%25c3%25b4pitals-rule-how-solve-this-limit-question%23new-answer', 'question_page');
                      }
                      );

                      Post as a guest




















































































                      Popular posts from this blog

                      Plaza Victoria

                      In PowerPoint, is there a keyboard shortcut for bulleted / numbered list?

                      How to put 3 figures in Latex with 2 figures side by side and 1 below these side by side images but in...